Second chance LR questions

¡Supera tus tareas y exámenes ahora con Quizwiz!

T59 - S2 - Q3 Despite the enormous number of transactions processed daily by banks nowadays, if a customer's bank account is accidentally credited with a large sum of money, it is extremely unlikely that the error will not be detected by the bank's internal audit procedures. Which one of the following, if true, most strongly supports the claim above? A Banks initially process all transactions using one set of computer programs, but then use a different set of programs to double-check large transactions. B Recent changes in banking standards require that customers present identification both when making deposits into their accounts and when making withdrawals from their accounts. C Banks are required by law to send each customer a monthly statement detailing every transaction of the previous month. D The average ratio of bank auditors to customer accounts has slowly increased over the past 100 years. E The development of sophisticated security software has rendered bank computers nearly impervious to tampering by computer hackers.

A.

T58 - S1 - Q11 Recent studies indicate a correlation between damage to human chromosome number six and adult schizophrenia. We know, however, that there are people without damage to this chromosome who develop adult schizophrenia and that some people with damage to chromosome number six do not develop adult schizophrenia. So there is no causal connection between damage to human chromosome number six and adult schizophrenia. Which one of the following most accurately describes a reasoning flaw in the argument above? A The argument ignores the possibility that some but not all types of damage to chromosome number six lead to schizophrenia. B The argument presumes, without providing evidence, that schizophrenia is caused solely by chromosomal damage. C The argument makes a generalization based on an unrepresentative sample population. D The argument mistakes a cause for an effect. E The argument presumes, without providing warrant, that correlation implies causation.

A. Correlation doesn't prove causation, it's true. But the lack of perfect correlation doesn't disprove causation, either. This dude's argument is destroyed. We're asked to find the flaw. My prediction is "assumes that if a correlation isn't perfect there can't be causation." A will do it. This is like saying "certain genetically-predisposed people will get cancer from smoking," or "certain gunshots to the face will cause death"—maybe some types of damage to chromosome six cause schizophrenia even if all types of damage to that chromosome do not. Even still, there can be a causal relationship between damage to the chromosome and schizophrenia. This is a perfect answer.

T57 - S2 - Q15 Carla: Professors at public universities should receive paid leaves of absence to allow them to engage in research. Research not only advances human knowledge, but also improves professors' teaching by keeping them abreast of the latest information in their fields. David: But even if you are right about the beneficial effects of research, why should our limited resources be devoted to supporting professors taking time off from teaching? David's response to Carla is most vulnerable to criticism on the grounds that it A ignores the part of Carla's remarks that could provide an answer to David's question B takes for granted that the only function of a university professor is teaching C incorrectly takes Carla's remarks as claiming that all funding for professors comes from tax money D takes for granted that providing the opportunity for research is the only function of paid leaves of absence E presumes, without providing justification, that professors do not need vacations

A. David is a *******. Carla says professors should get paid leaves of absence to allow them to engage in research because this advances human knowledge and furthermore improves professors' teaching. David, picking his nose, then goes, "Duh... why should we pay teachers not to teach?" Put yourself in Carla's shoes and get indignant! "David, you jackass, did you not listen to what I just said? I told you that paying them for time off will improve their teaching." If you can get pissed off, it will make the questions a whole lot easier. If you're not outraged, you're not paying attention. A) David simply wasn't listening; Carla already answered his question.

T59 - S2 - Q15 Bowers: A few theorists hold the extreme view that society could flourish in a condition of anarchy, the absence of government. Some of these theorists have even produced interesting arguments to support that position. One writer, for example, contends that anarchy is laissez-faire capitalism taken to its logical extreme. But these theorists' views ignore the fundamental principle of social philosophy—that an acceptable social philosophy must promote peace and order. Any social philosophy that countenances chaos, i.e., anarchy, accordingly deserves no further attention. The reasoning in Bowers's argument is most vulnerable to criticism on the grounds that A the meaning of a key term shifts illicitly during the course of the argument B the argument fails to show that laissez-faire capitalism deserves to be rejected as a social philosophy C the truth or falsity of a view is not determined by the number of people who accept it as true D the argument presumes, without providing justification, that any peaceful society will flourish E it is unreasonable to reject a view merely because it can be described as extreme

A. Hold on a second. Who says that anarchy entails chaos? Who even says that anarchy and chaos are the same things? I don't think the theorists mentioned in the first sentence would agree that chaos=anarchy. They defined anarchy as "the absence of government." So, we can't dismiss their argument unless we're playing by their terms. Otherwise, we're just talking past each other; we're not genuinely arguing. This is a Flaw question. I'll tell you right now, the flaw is that the meaning of anarchy changes from the "absence of government," which could be a perfectly orderly society, to "chaos," which might have nothing to do with an absence of government. Case in point, look at the Trump administration. There's a government with a fair bit of chaos. It's A, as predicted. How the theorists define anarchy and how the author defines anarchy in the last sentence are not necessarily synonymous.

T59 - S2 - Q19 At a gathering at which bankers, athletes, and lawyers are present, all of the bankers are athletes and none of the lawyers are bankers. If the statements above are true, which one of the following statements must also be true? A All of the athletes are bankers. B Some of the lawyers are not athletes. C Some of the athletes are not lawyers. D All of the bankers are lawyers. E None of the lawyers are athletes.

C. "Bankers" is mentioned in both premises, so let's see if we can combine those premises together. All bankers are athletes. No banker is a lawyer. All of the bankers at the party are 1) athletes and 2) not lawyers. So there has to be an overlap between "athlete" and "not a lawyer." It's a Must Be True, so one of the answers will be 100% proven by the facts on the page. Four wrong answers will be different or extra. There's an overlap between "athlete" and "non-lawyer." Because of the bankers, I know for sure that there are some athletes at the party who are not lawyers.

T50 - S2 - Q24 Columnist: The amount of acidic pollutants released into the air has decreased throughout the world over the last several decades. We can expect, then, an overall decrease in the negative environmental effects of acid rain, which is caused by these acidic pollutants. Each of the following, if true, would weaken the columnist's argument EXCEPT: A Some ecosystems have developed sophisticated mechanisms that reduce the negative effects of increased levels of acids in the environment. B The amount of acid-neutralizing buffers released into the air has decreased in recent years. C The current decrease in acidic pollutants is expected to end soon, as more countries turn to coal for the generation of electricity. D The effects of acid rain are cumulative and largely independent of current acid rain levels. E The soils of many ecosystems exposed to acid rain have been exhausted of minerals that help protect them from acid rain's harmful effects.

A. If we were to attack the columnist's argument—and we should always try our hardest to do so—we might say, "Well, aren't there potentially many causes of acid rain? Perhaps 'acidic pollutants' is only one cause. Reducing that single cause might not reduce all causes. Furthermore, even if acid rain were totally eliminated, how do you know that the negative effects of past acid rain won't still be around? For example, perhaps last year's acid rain is now trapped in our snowpacks and polar icecaps, where it doesn't currently harm us, but where it is going to ruin us in a few years when it melts?" The question asks us to identify four weakeners, and to choose the one correct answer that does not weaken. So the correct answer could strengthen the argument, or it could just be irrelevant. Let's see: A) could only possibly strengthen the conclusion that the negative effects of acid rain are going to decrease. So this is probably the answer.

T58 - S1 - Q5 Doctor: While a few alternative medicines have dangerous side effects, some, such as many herbs, have been proven safe to consume. Thus, though there is little firm evidence of medicinal effect, advocates of these herbs as remedies for serious illnesses should always be allowed to prescribe them, since their patients will not be harmed, and might be helped, by the use of these products. Which one of the following, if true, most seriously weakens the doctor's argument? A Many practitioners and patients neglect more effective conventional medicines in favor of herbal remedies. B Many herbal remedies are marketed with claims of proven effectiveness when in fact their effectiveness is unproven. C Some patients may have allergic reactions to certain medicines that have been tolerated by other patients. D The vast majority of purveyors of alternative medicines are driven as much by the profit motive as by a regard for their patients' health. E Any pain relief or other benefits of many herbs have been proven to derive entirely from patients' belief in the remedy, rather than from its biochemical properties.

A. It turns out to be a Weaken question, but it could just as easily have been a Strengthen question—we can win the case for either side; it's just a matter of who cuts us a retainer check first: Weaken: Patients might use herbal remedies in lieu of life-saving therapies that actually work. Strengthen: Patients would never use an herbal remedy in lieu of life-saving therapies that actually work. Or it could have been an assumption question: Necessary Assumption: At least one patient can be helped, and not harmed, by an herbal remedy. Sufficient Assumption: No patient can be harmed by an herbal remedy, and doctors should always be allowed to prescribe things that cause no harm. Or it could have been a Flaw question, or a Reasoning question: Flaw: The argument has failed to consider the possibility that patients prescribed herbal remedies might forego life-saving treatments that actually work. Reasoning: The argument claimed that, because a certain course of action has no direct detrimental effects, it cannot thereby cause overall harm. If A is true, the argument is destroyed. Cilantro can't cause harm directly, but it can kill you if you take cilantro instead of life-saving therapy.

T60 - S1 - Q22 One of the most useful social conventions is money, whose universality across societies is matched only by language. Unlike language, which is rooted in an innate ability, money is an artificial, human invention. Hence, it seems probable that the invention of money occurred independently in more than one society. The argument's conclusion is properly drawn if which one of the following is assumed? A Some societies have been geographically isolated enough not to have been influenced by any other society. B Language emerged independently in different societies at different times in human history. C Universal features of human society that are not inventions are rooted in innate abilities. D If money were not useful, it would not be so widespread. E No human society that adopted the convention of money has since abandoned it.

A. One way to bridge the gap from facts to conclusion is, "Anything that is universal across societies but is not innate was probably invented independently in more than one society." Another, equally good, answer would be, "Anything that is universal across societies but is an artificial human invention was probably invented independently in more than one society." All societies have money. If there are some (one or more) societies that have never been influenced by any other society then they must have invented money independently. If they are so geographically isolated, it "seems probable" that they didn't influence other societies either. In that case, it "seems probable that the invention of money occurred independently in more than one society."

T60 - S1 - Q17 Humankind would not have survived, as it clearly has, if our ancestors had not been motivated by the desire to sacrifice themselves when doing so would ensure the survival of their children or other close relatives. But since even this kind of sacrifice is a form of altruism, it follows that our ancestors were at least partially altruistic. Which one of the following arguments is most similar in its reasoning to the argument above? A Students do not raise their grades if they do not increase the amount of time they spend studying. Increased study time requires good time management. However, some students do raise their grades. So some students manage their time well. B Organisms are capable of manufacturing their own carbohydrate supply if they do not consume other organisms to obtain it. So plants that consume insects must be incapable of photosynthesis, the means by which most plants produce their carbohydrate supplies. C If fragile ecosystems are not protected by government action their endemic species will perish, for endemic species are by definition those that exist nowhere else but in those ecosystems. D The natural resources used by human beings will be depleted if they are not replaced by alternative materials. But since such replacement generally requires more power, the resources used to create that power will become depleted. E Public buildings do not harmonize with their surroundings if they are not well designed. But any well-designed building is expensive to construct. Thus, either public buildings are expensive to construct or else they do not harmonize with their surroundings.

A. Premise: Our ancestors survived. Premise: Their survival required sacrifices on behalf of children and other close relatives. Premise: Sacrifices mean altruism. Conclusion: Our ancestors were at least partially altruistic. Those premises, if true, prove the conclusion correct. Heading into the answer choices, I'm looking for a valid argument. Specifically, I'd love something in the form of "X happened, and it required Y. Y is a form of Z, therefore X implies Z." Premise: Some kids raised their grades. Premise: Raising your grades requires increased study time. Premise: Increased study time means good time management. Conclusion: So some kids managed their time well. Notice that the elements in this answer were in slightly different order than they were in the given argument. The given argument said "our ancestors survived" right up top. This answer said "some kids raised their grades" near the bottom. This doesn't matter in the slightest. The order of the elements in an argument don't change its logical structure. The pieces are all the same. So this is the answer.

T60 - S3 - Q22 Consumer advocate: There is no doubt that the government is responsible for the increased cost of gasoline, because the government's policies have significantly increased consumer demand for fuel, and as a result of increasing demand, the price of gasoline has risen steadily. Which one of the following is an assumption required by the consumer advocate's argument? A The government can bear responsibility for that which it indirectly causes. B The government is responsible for some unforeseen consequences of its policies. C Consumer demand for gasoline cannot increase without causing gasoline prices to increase. D The government has an obligation to ensure that demand for fuel does not increase excessively. E If the government pursues policies that do not increase the demand for fuel, gasoline prices tend to remain stable.

A. Premise: The price of gas has risen steadily. Premise: Increasing demand has caused this price increase. Premise: The government has increased consumer demand for fuel. Conclusion: The government is responsible for the increased cost of gasoline. The logic actually seems pretty sound to me. The government made a policy, and the policy had result A, which in turn had another result, B. Therefore the government can be called "responsible" for B. We're asked to find a necessary assumption of the argument. In other words, we need to find an answer that, if untrue, causes the argument to fail. If A is untrue, it becomes, "The government cannot bear responsibility for that which it indirectly causes." The government didn't directly cause the fuel price increase; it did so indirectly, by stimulating consumer demand. So if the government can't bear responsibility for something it indirectly caused, the government can't be responsible for the fuel price increase. This means answer A, "The government can bear responsibility for that which it indirectly causes," is a necessary assumption of the argument.

T54 - S2 - Q24 There are 1.3 billion cows worldwide, and this population is growing to keep pace with the demand for meat and milk. These cows produce trillions of liters of methane gas yearly, and this methane contributes to global warming. The majority of the world's cows are given relatively low-quality diets even though cows produce less methane when they receive better-quality diets. Therefore, methane production from cows could be kept in check if cows were given better-quality diets. Which one of the following, if true, adds the most support for the conclusion of the argument? A Cows given good-quality diets produce much more meat and milk than they would produce otherwise. B Carbon and hydrogen, the elements that make up methane, are found in abundance in the components of all types of cow feed. C Most farmers would be willing to give their cows high-quality feed if the cost of that feed were lower. D Worldwide, more methane is produced by cows raised for meat production than by those raised for milk production. E Per liter, methane contributes more to global warming than does carbon dioxide, a gas that is thought to be the most significant contributor to global warming.

A. So my objection to the logic is this: If the population of cows keeps growing to keep pace with human demand, might the methane problem also keep growing, even if we give them a better diet? A better diet might reduce methane per cow, but would do nothing to affect the number of cows. And the number of cows seems to be the real problem here. A, is a solid answer. Not only do better diets reduce methane per cow, they also increase meat and milk production per cow, thus decreasing the number of cows required to support the human race's irresponsible population growth. This defends against my predicted weakener, so I like this answer.

T59 - S2 - Q22 A recent study confirms that nutritious breakfasts make workers more productive. For one month, workers at Plant A received free nutritious breakfasts every day before work, while workers in Plant B did not. The productivity of Plant A's workers increased, while that of Plant B's workers did not. Which one of the following, if true, most strengthens the argument? A Few workers in Plant B consumed nutritious breakfasts during the month of the study. B Workers in the study from Plant A and Plant B started work at the same time of day. C During the month before the study, workers at Plant A and Plant B were equally productive. D Workers from Plant A took fewer vacation days per capita during the month than did workers from Plant B. E Workers in Plant B were more productive during the month of the study than were workers from Plant A.

A. The workers at Plant B may not have received a free breakfast, but does that guarantee that they weren't eating a nutritious breakfast independently? This study leaves open the possibility that the laborers at Plant B made food for themselves. If that happened, then the study doesn't prove that nutritious breakfast makes workers more productive. We need to find a way to strengthen this argument. The perfect strengthener would be a statement saying that workers at Plant B did not obtain a nutritious breakfast. Some workers at Plant B might have consumed a nutritious breakfast during the course of the study, but most of them didn't. A, patches up the argument's weakness, so it's a strengthener.

T58 - S1 - Q25 If a wig has any handmade components, it is more expensive than one with none. Similarly, a made-to-measure wig ranges from medium-priced to expensive. Handmade foundations are never found on wigs that do not use human hair. Furthermore, any wig that contains human hair should be dry-cleaned. So all made-to-measure wigs should be dry-cleaned. The conclusion of the argument follows logically if which one of the following is assumed? A Any wig whose price falls in the medium-priced to expensive range has a handmade foundation. B If a wig's foundation is handmade, then it is more expensive than one whose foundation is not handmade. C A wig that has any handmade components should be dry-cleaned. D If a wig's foundation is handmade, then its price is at least in the medium range. E Any wig that should be dry-cleaned has a foundation that is handmade.

A. This missing connection, when made, causes the machine to work. So my prediction is "if you're medium-priced or expensive, you have a handmade foundation." I'll be awful surprised if that's not the answer.

T58 - S1 - Q26 Philosopher: Wolves do not tolerate an attack by one wolf on another if the latter wolf demonstrates submission by baring its throat. The same is true of foxes and domesticated dogs. So it would be erroneous to deny that animals have rights on the grounds that only human beings are capable of obeying moral rules. The philosopher's argument proceeds by attempting to A provide counterexamples to refute a premise on which a particular conclusion is based B establish inductively that all animals possess some form of morality C cast doubt on the principle that being capable of obeying moral rules is a necessary condition for having rights D establish a claim by showing that the denial of that claim entails a logical contradiction E provide evidence suggesting that the concept of morality is often applied too broadly

A. Wait what? This argument assumes that wolves and foxes and dogs are "obeying moral rules" when they display a certain behavior. That's a pretty big leap. Furthermore, since when are only creatures "capable of obeying moral rules" possessors of rights? The question basically a Must Be True. Which one of the following did the argument definitely do? Yeah, I guess wolves, foxes, and dogs are the "counterexamples" to the premise "only human beings are capable of obeying moral rules." The "particular conclusion" is "it would be erroneous to deny that animals have rights." This seems like a good fit.

T60 - S3 - Q20 Principle: One should criticize the works or actions of another person only if the criticism will not seriously harm the person criticized and one does so in the hope or expectation of benefiting someone other than oneself. Application: Jarrett should not have criticized Ostertag's essay in front of the class, since the defects in it were so obvious that pointing them out benefited no one. Which one of the following, if true, justifies the above application of the principle? A Jarrett knew that the defects in the essay were so obvious that pointing them out would benefit no one. B Jarrett's criticism of the essay would have been to Ostertag's benefit only if Ostertag had been unaware of the defects in the essay at the time. C Jarrett knew that the criticism might antagonize Ostertag. D Jarrett hoped to gain prestige by criticizing Ostertag. E Jarrett did not expect the criticism to be to Ostertag's benefit.

A. We're asked to "justify," or "make correct," the application of the principle. If I were going to put some evidence into the record, I would want to show that Jarrett could not possibly have thought he was being helpful. Like, what if we had a witness who talked to Jarrett right before class, to whom Jarrett had said, "I know this isn't going to benefit anyone, but I'm gonna say it anyway," or, "I'm going to kiss ass to the teacher by criticizing Ostertag....that's the *only* reason I would say anything." If either of these is true, then Jarrett was not making his comment in order to benefit anyone other than himself, which would mean that his criticism was unjustified. If Jarrett *knew* he wasn't helping anyone, then he should not have criticized Ostertag.

T46 - S2 - Q20 Professor Donnelly's exams are always more difficult than Professor Curtis's exams. The question about dinosaurs was on Professor Donnelly's last exam. Therefore, the question must be difficult. Which one of the following exhibits both of the logical flaws exhibited in the argument above? A Lewis is a better baker than Stockman. Lewis made this cake. Therefore, it must be better than most of Stockman's cakes. B Porter's new book of poetry is better than any of her other books of poetry. This poem is from Porter's new book, so it must be good. C Professor Whitburn is teaching English this year and always assigns a lot of reading. Therefore, this year's English class will have to do more reading than last year's class. D Shield's first novel has a more complicated plot than any other that she has written. Hence, that plot must be very complex. E Mathematics is more difficult than history. Therefore, my calculus test will be more difficult than my history test.

B.

T59 - S2 - Q24 Market analyst: According to my research, 59 percent of consumers anticipate paying off their credit card balances in full before interest charges start to accrue, intending to use the cards only to avoid carrying cash and writing checks. This research also suggests that in trying to win business from their competitors, credit card companies tend to concentrate on improving the services their customers are the most interested in. Therefore, my research would lead us to expect that______________. Which one of the following most logically completes the market analyst's argument? A most consumers would be indifferent about which company's credit card they use B credit card companies would not make the interest rates they charge on cards the main selling point C most consumers would prefer paying interest on credit card debts over borrowing money from banks D most consumers would ignore the length of time a credit card company allows to pay the balance due before interest accrues E the most intense competition among credit card companies would be over the number of places that they can get to accept their credit card

B. Apparently, 59% of all consumers plan to follow this advice. If this is true, and if credit card companies tend to emphasize features that their customers would be most interested in, which one of the following must be true? B is 100% proven true by the given facts. Remember, "in trying to win business from their competitors, credit card companies tend to concentrate on improving the services their customers are the most interested in." If more than 50% of all consumers plan to never pay interest on their cards, credit card companies would be idiots to focus on their interest rates. They can focus on literally any other feature, but they can't focus on rates if the given facts are true.

T57 - S2 - Q26 Historian: Flavius, an ancient Roman governor who believed deeply in the virtues of manual labor and moral temperance, actively sought to discourage the arts by removing state financial support for them. Also, Flavius was widely unpopular among his subjects, as we can conclude from the large number of satirical plays that were written about him during his administration. The historian's argumentation is most vulnerable to criticism on the grounds that it A fails to consider the percentage of plays written during Flavius's administration that were not explicitly about Flavius B treats the satirical plays as a reliable indicator of Flavius's popularity despite potential bias on the part of the playwrights C presumes, without providing evidence, that Flavius was unfavorably disposed toward the arts D takes for granted that Flavius's attempt to discourage the arts was successful E fails to consider whether manual labor and moral temperance were widely regarded as virtues in ancient Rome

B. Nah, this is bullshit because the conclusion is much broader than what's warranted by the facts. Flavius cut funding for the arts, so it's not a shocker that he was criticized in many plays, which are, after all, part of the arts. It would be foolish to assume, on these facts, that Flavius was "widely unpopular among his subjects," unless of course his subjects were all artists, which is not in evidence here. We're asked to find a flaw in the logic. One way of stating it would be "makes a broader conclusion than is warranted by the facts." Playwrights might have hated Flavius for cutting their paychecks, so the opinion of playwrights isn't a "reliable indicator" of his popularity among his subjects more broadly. B is a great answer.

T50 - S2 - Q9 One should always capitalize the main words and the first and last words of a title. But one should never capitalize articles, or prepositions and conjunctions with fewer than five letters, when they occur in the middle of a title. Which one of the following can be properly inferred from the statements above? A If a word that is a preposition or conjunction should be capitalized, then it is the first or last word of the title. B If a word in the middle of a title should be capitalized, then that word is neither an article nor a conjunction shorter than five letters. C All prepositions and conjunctions with fewer than five letters should be uncapitalized in titles. D If a word is neither a main word nor a first or last word of a title, then it should not be capitalized. E Prepositions and conjunctions with five or more letters should be capitalized in any text.

B. The correct answer (B) is a weaker, easier to prove version of all that. It's a Must Be True / Reasoning question, so we prefer a weaker answer anyway. If my prediction is true then B is definitely true (at least.)

T46 - S2 - Q7 Although instinct enables organisms to make complex responses to stimuli, instinctual behavior involves no reasoning and requires far fewer nerve cells than does non-instinctual (also called flexible) behavior. A brain mechanism capable of flexible behavior must have a large number of neurons, and no insect brain has yet reached a size capable of providing a sufficiently large number of neurons. Which one of the following can be properly inferred from the statements above? A The behavior of organisms with elaborate brain mechanisms is usually not instinctual. B Insect behavior is exclusively instinctual. C All organisms with brains larger than insects' brains are capable of some measure of flexible behavior. D All organisms with large brains are biologically equipped for flexible behavior. E Only organisms with brains of insect size or smaller engage in purely instinctual behavior.

B. The first sentence says there's two types of behavior. There's instinctive behavior, which involves no reasoning and fewer nerve cells than the other alternative, non-instinctive behavior. The second sentence says that this non-instinctive behavior requires lots of neurons, and adds that no insect brain has yet reached a size that's capable of providing this many neurons. From these statements alone, with no outside help or knowledge, we can infer that insects aren't capable of non-instinctive behavior. Duh, right? I mean, what else could this speaker possibly have come here to tell us if not "insects can't do non-instinctive behavior"? We're looking for an answer that's been proven by the facts on the page. B) Yup. Insects simply aren't capable of non-instinctual behavior. So their behavior must be non-instinctual. Some students will object here, asking "but can't there be a third type of behavior? It never says that 'instinctual' and 'non-instinctual' are the only two types of behaviors!" I'm glad you're being critical of the answer choices, but there do exist binary conditions in the world, and I think "instinctual" and "non-instinctual" fall pretty easily into the "binary" category. If someone said you have a choice between an alcoholic and a nonalcoholic beer, would you really be asking "what about other types of beers... I want one that's not alcoholic, but also one that's not not alcoholic"? If you were asked to check a box between "married" and "unmarried," would you really say "what if I'm neither"?

T60 - S3 - Q24 Music critic: How well an underground rock group's recordings sell is no mark of that group's success as an underground group. After all, if a recording sells well, it may be because some of the music on the recording is too trendy to be authentically underground; accordingly, many underground musicians consider it desirable for a recording not to sell well. But weak sales may simply be the result of the group's incompetence. Which one of the following principles, if valid, most helps to justify the music critic's argument? A If an underground rock group is successful as an underground group, its recordings will sell neither especially well nor especially poorly. B An underground rock group is unsuccessful as an underground group if it is incompetent or if any of its music is too trendy to be authentically underground, or both. C Whether an underground group's recordings meet criteria that many underground musicians consider desirable is not a mark of that group's success. D An underground rock group is successful as an underground group if the group is competent but its recordings nonetheless do not sell well. E For an underground rock group, competence and the creation of authentically underground music are not in themselves marks of success.

B. The flaw in this argument is that there is no explicit definition of what "success" means. We might make the inference that because being too trendy or incompetent is a failure, that being underground and competent means success, but this is only an inference. We should search for an answer choice that makes this relationship explicit. B makes sense and closes the gap we identified in the stimulus. Failure could come from being bad or incompetent or too mainstream, and thus, not underground. This answer makes explicit an assumption that underlies the stimulus: that being too popular is a potential sign of failure in the underground project.

T54 - S2 - Q13 Consumers seek to purchase the highest quality at the lowest prices. Companies that do not offer products that attract consumers eventually go bankrupt. Therefore, companies that offer neither the best quality nor the lowest price will eventually go bankrupt. The conclusion above follows logically if which one of the following is assumed? A No company succeeds in producing a product that is both highest in quality and lowest in price. B Products that are neither highest in quality nor lowest in price do not attract consumers. C Any company that offers either the highest quality or the lowest price will avoid bankruptcy. D Some consumers will not continue to patronize a company purely out of brand loyalty. E No company is driven from the market for reasons other than failing to meet consumer demands.

B. This argument ignores the middle ground. Basically it says customers want cheap, and they want quality. So if you're not either Wal-Mart (cheap) or Apple (quality), you're going bankrupt. My objection is: What about Costco? They offer very nice stuff (but not the best), at very fair prices (but not the absolute cheapest). The question then asks us to prove the argument. So the answer choice needs to eliminate the middle ground. I'm looking for something that would mean Costco, in my example above, doesn't exist. B would do it, because if it's true then nobody would shop at Costco. Every-one would go to either Wal-Mart (cheapest) or Apple (the best).

T59 - S2 - Q26 Commentator: For a free market to function properly, each prospective buyer of an item must be able to contact a large number of independent prospective sellers and compare the prices charged for the item to what the item is worth. Thus, despite advertised prices and written estimates available from many of its individual businesses, the auto repair industry does not constitute a properly functioning free market. The conclusion of the commentator's argument follows logically if which one of the following is assumed? A People do not usually shop for auto repairs but instead take their autos to their regular repair shop out of habit. B Some persons who are shopping for auto repairs cannot determine what these repairs are worth. C Not all auto repair shops give customers written estimates. D Many auto repair shops charge more for auto repairs than these repairs are worth. E Because it is not regulated, the auto repair industry does not have standardized prices.

B. We are asked to find an assumption that allows the conclusion to "follow logically." In other words, we are asked to be the attorney for the Commentator, and come up with a dream premise that will prove the Commentator's conclusion. This is a Sufficient Assumption question. Pretend you're going to hire an expert witness on behalf of the Commentator. An expert witness, if you have enough money, will say whatever you want her to say. If you need to prove that the auto repair industry is not a properly functioning free market, what do you want your expensive expert to say? Think about it for a minute. It's very simple. You must connect the evidence you already have to the conclusion the Commentator is trying to make. You should be able to predict the correct answer here with frightening certainty. My prediction is, "The auto repair industry does not allow each prospective buyer to contact a large number of independent prospective sellers and compare prices." As we go through the answer choices, let's pretend each one is a different prospective expert witness. Which one are we going to hire? Even though B isn't exactly what we predicted, I do think that if it is true then the auto repair market can't possibly be a "properly functioning free market" as defined by the Commentator.

In an experiment, researchers played a series of musical intervals—two-note sequences—to a large, diverse group of six-month-old babies. They found that the babies paid significantly more attention when the intervals were perfect octaves, fifths, or fourths than otherwise. These intervals are prevalent in the musical systems of most cultures around the world. Thus, humans probably have a biological predisposition to pay more attention to those intervals than to others. Which one of the following, if true, most strengthens the argument? A Several similar experiments using older children and adults found that these subjects, too, had a general tendency to pay more attention to octaves, fifths, and fourths than to other musical intervals. B None of the babies in the experiment had previous exposure to music from any culture. C All of the babies in the experiment had been exposed to music drawn equally from a wide variety of cultures around the world. D In a second experiment, these same babies showed no clear tendency to notice primary colors more than other colors. E Octaves, fifths, and fourths were played more frequently during the experiment than other musical intervals were.

B. We're asked to strengthen the argument, so my first guess is that the correct answer will plug the hole we have identified. Something like, "It's impossible to learn to pay attention to perfect octaves," would plug the gap nicely. Let's see. Okay, B is good. If this is true, then the babies in the study couldn't have been indoctrinated. Instead, they're naturally paying attention to perfect octaves. I like this answer.

T58 - S1 - Q13 It is a given that to be an intriguing person, one must be able to inspire the perpetual curiosity of others. Constantly broadening one's abilities and extending one's intellectual reach will enable one to inspire that curiosity. For such a perpetual expansion of one's mind makes it impossible to be fully comprehended, making one a constant mystery to others. Which one of the following most accurately expresses the conclusion drawn in the argument above? A To be an intriguing person, one must be able to inspire the perpetual curiosity of others. B If one constantly broadens one's abilities and extends one's intellectual reach, one will be able to inspire the perpetual curiosity of others. C If one's mind becomes impossible to fully comprehend, one will always be a mystery to others. D To inspire the perpetual curiosity of others, one must constantly broaden one's abilities and extend one's intellectual reach. E If one constantly broadens one's abilities and extends one's intellectual reach, one will always have curiosity.

B. Yes, this answer paraphrases the second sentence, which was the main conclusion. The author said that constantly broadening your abilities will lead you to inspire curiosity. That establishes a conditional relationship, which is why this answer could be paraphrased in a classic "if...then" format.

T60 - S1 - Q15 Camille: Manufacturers of water-saving faucets exaggerate the amount of money such faucets can save. Because the faucets handle such a low volume of water, people using them often let the water run longer than they would otherwise. Rebecca: It is true that showering now takes longer. Nevertheless, I have had lower water bills since I installed a water-saving faucet. Thus, it is not true that the manufacturers' claims are exaggerated. The reasoning in Rebecca's argument is questionable in that she takes for granted that: A the cost of installing her water-saving faucet was less than her overall savings on her water bill B she saved as much on her water bills as the manufacturers' claims suggested she would C the manufacturers' claims about the savings expected from the installation of water-saving faucets are consistent with one another D people who use water-saving faucets are satisfied with the low volume of water handled by such faucets E installing more water-saving faucets in her house would increase her savings

B. Camille's claim of exaggeration could still be true even if the water bill is lower. Maybe the manufacturers claim that you will save 10 percent of your money, but because people run the faucets longer, they only save 5. Just because the bill is lower does not mean there was no exaggeration.

T46 - S2 - Q22 Detective: Laser-printer drums are easily damaged, and any nick in a drum will produce a blemish of similar dimensions on each page produced by that printer. So in matching a blemish on a page with a nick on a drum, we can reliably trace a suspicious laser-printed document to the precise printer on which it was produced. Which one of the following, if true, most weakens the detective's argument? A Criminals are unlikely to use their own laser printers to produce suspicious documents. B Drum nicks are usually so small that it requires skill to accurately determine their size and shape. C The manufacturing process often produces the same nick on several drums. D Blemishes on documents are sometimes totally concealed by characters that are printed over them. E Most suspicious documents are not produced on laser printers.

C.

T50 - S2 - Q6 Classical Roman architecture is beautiful, primarily because of its use of rounded arches and its symmetry. Postmodern architecture is dramatic, primarily because of its creative use both of materials and of the surrounding environment. An architectural style that combines elements of both classical Roman and postmodern architecture would therefore be both beautiful and dramatic. The reasoning in the argument is flawed in that it A presumes, without providing justification, that for an architectural style to have certain qualities, its components must have those qualities B fails to justify its presumption that because postmodern architecture is dramatic, that is its most salient feature C neglects to consider that an architectural style combining elements of two other architectural styles may lack certain qualities of one or both of those styles D neglects to specify how the drama of an architectural style contributes to its beauty E ignores the possibility that there are other architectural styles whose defining qualities include both drama and beauty

C.

T57 - S2 - Q13 Administrators of educational institutions are enthusiastic about the educational use of computers because they believe that it will enable schools to teach far more courses with far fewer teachers than traditional methods allow. Many teachers fear computers for the same reason. But this reason is mistaken. Computerized instruction requires more, not less, time of instructors, which indicates that any reduction in the number of teachers would require an accompanying reduction in courses offered. The statement that the educational use of computers enables schools to teach far more courses with far fewer teachers figures in the argument in which one of the following ways? A It is presented as a possible explanation for an observation that follows it. B It is a statement of the problem the argument sets out to solve. C It is a statement that the argument is designed to refute. D It is a statement offered in support of the argument's main conclusion. E It is the argument's main conclusion.

C.

T47 - S1 - Q24 Consumer advocate: The manufacturer's instructions for assembling a product should be written in such a way that most consumers would find it much easier to put the product together if the instructions were available than if they were not. Which one of the following, if true, would provide the strongest reason for thinking that the principle advanced by the consumer advocate cannot always be followed? A The typical consumer who assembles a product does so using the manufacturer's instructions, but still has great difficulty. B Often the store at which a consumer purchases an unassembled product will offer, for a fee, to assemble the product and deliver it. C For the typical product, most consumers who assemble it do so very easily and without ever consulting the manufacturer's instructions. D Usually a consumer who is trying to assemble a product using the manufacturer's instructions has no difficulty understanding the instructions. E Some consumers refer to the manufacturer's instructions for assembling a product only if they have difficulty assembling the product.

C. I think that the key here is "most" in the passage. It's saying that instructions should make assembling the product "much easier" for "most" consumers than no instructions at all. In other words, that's one way of testing whether instructions are good. If they make it "much easier" for "most" consumers to put something together, then they are good—or at least written in a way that they "should" be. We're just looking for an answer that suggests that you can't always follow this rule. Which answer suggests that you can't always write instructions that make it "much easier" for "most" consumers to put something together? Answer C is correct because if "most" consumers never even consult the instructions, then the instructions would never make it easier for "most" consumers. They might make it easier for "some" consumers—the ones who do consult the instructions. But the test was whether "most" consumers found it "much easier" with the instructions.

T60 - S1 - Q16 Company spokesperson: In lieu of redesigning our plants, our company recently launched an environmental protection campaign to buy and dispose of old cars, which are generally highly pollutive. Our plants account for just 4 percent of the local air pollution, while automobiles that predate 1980 account for 30 percent. Clearly, we will reduce air pollution more by buying old cars than we would by redesigning our plants. Which one of the following, if true, most seriously weakens the company spokesperson's argument? A Only 1 percent of the automobiles driven in the local area predate 1980. B It would cost the company over $3 million to reduce its plants' toxic emissions, while its car-buying campaign will save the company money by providing it with reusable scrap metal. C Because the company pays only scrap metal prices for used cars, almost none of the cars sold to the company still run. D Automobiles made after 1980 account for over 30 percent of local air pollution. E Since the company launched its car-buying campaign, the number of citizen groups filing complaints about pollution from the company's plants has decreased.

C. If almost all of the cars that the company ends up buying do not run, they are not reducing air pollution, because cars that do not run generally do not produce a lot of air pollution. Successfully weakened!

T47 - S1 - Q26 Researchers gave 100 first-graders after-school lessons in handwriting. They found that those whose composition skills had improved the most had learned to write letters the most automatically. This suggests that producing characters more automatically frees up mental resources for other activities. Which one of the following, if true, most strengthens the argument? A Among the first-graders who received the after-school lessons in handwriting, those who practiced the most learned to write letters the most automatically. B The first-graders who wrote letters the most automatically before receiving the after-school lessons in handwriting showed the greatest improvement in their composition skills over the course of the lessons. C Over the course of the lessons, the first-graders who showed greater improvement in their ability to write letters automatically also generally showed greater improvement in their composition skills. D Before receiving the after-school lessons in handwriting, the 100 first-graders who received the lessons were representative of first-graders more generally, with respect to their skills in both handwriting and composition. E Among the first-graders who received the lessons in handwriting, those who started out with strong composition skills showed substantial improvement in how automatically they could write letters.

C. On this one, I just have to hang in there and believe them when they say that, in their study of first-graders, those who improved composition the most also learned to write the most automatically. From there, they try to conclude, "this suggests that producing characters more automatically frees up mental resources for other activities." This is bizarre though. What if I had said, "the students who improved most at math also improved the most at English"? Would it make any sense if I then said: "so, improving at English helps you improve at math?" Or do you have another explanation? I sure do. How about "the students who study the hardest improved at both math and English because they're the hardest workers," or "the students who are just plain smarter / better learners improved at both subjects the most, because they're the best learners?" See the argument's big weakness? Good. Now switch teams and find a strengthener. Something that defends it against attacks on its glaring weakness would be good. C) shows that the phenomenon of "more automatic writing, better composition" holds true across all levels of student. Even if you're at the bottom of the class in both of these things, when you improve one you tend to improve the other. That removes the "argument from extremes" problem with the given argument. Note that it still doesn't prove causation, but at least it establishes a correlation instead of one data point about the top kids in the class.

T58 - S1 - Q21 Columnist: Although there is and should be complete freedom of thought and expression, that does not mean that there is nothing wrong with exploiting depraved popular tastes for the sake of financial gain. Which one of the following judgments conforms most closely to the principle cited by the columnist? A The government should grant artists the right to create whatever works of art they want to create so long as no one considers those works to be depraved. B People who produce depraved movies have the freedom to do so, but that means that they also have the freedom to refrain from doing so. C There should be no laws restricting what books are published, but publishing books that pander to people with depraved tastes is not thereby morally acceptable. D The public has the freedom to purchase whatever recordings are produced, but that does not mean that the government may not limit the production of recordings deemed to be depraved. E One who advocates complete freedom of speech should not criticize others for saying things that he or she believes to exhibit depraved tastes.

C. The principle here is that 1. Freedom of thought and expression exists and is good, and 2. This freedom does not mean there isn't anything wrong with making money off of depraved things. C makes sense to me. You have a right to express whatever, but that doesn't mean it is morally okay.

T54 - S2 - Q26 The government will purchase and install new severe weather sirens for this area next year if replacement parts for the old sirens are difficult to obtain. The newspaper claims that public safety in the event of severe weather would be enhanced if new sirens were to be installed. The local company from which replacement parts were purchased last year has since gone out of business. So, if the newspaper is correct, the public will be safer during severe weather in the future. The argument's conclusion follows logically from its premises if which one of the following is assumed? A If public safety in the event of severe weather is enhanced next year, it will be because new sirens have been purchased. B The newspaper was correct in claiming that public safety in the event of severe weather would be enhanced if new sirens were purchased. C The local company from which replacement parts for the old sirens were purchased last year was the only company in the area that sold them. D Replacement parts for the old sirens will be difficult to obtain if the government cannot obtain them from the company it purchased them from last year. E Because the local company from which replacement parts had been purchased went out of business, the only available parts are of such inferior quality that use of them would make the sirens less reliable.

D.

T47 - S1 - Q22 Columnist: There are certain pesticides that, even though they have been banned for use in the United States for nearly 30 years, are still manufactured there and exported to other countries. In addition to jeopardizing the health of people in these other countries, this practice greatly increases the health risk to U.S. consumers, for these pesticides are often used on agricultural products imported into the United States. Which one of the following, if true, most seriously weakens the columnist's argument? A Trace amounts of some of the pesticides banned for use in the United States can be detected in the soil where they were used 30 years ago. B Most of the pesticides that are manufactured in the United States and exported are not among those banned for use in the United States. C The United States is not the only country that manufactures and exports the pesticides that are banned for use in the United States. D The banned pesticides pose a greater risk to people in the countries in which they are used than to U.S. consumers. E There are many pesticides that are banned for use in other countries that are not banned for use in the United States.

C. The story I tell myself here is, "The US bans DDT in the US, but exports DDT to Mexico, then imports tomatoes from Mexico that have the DDT on them." That fits with the given facts. The conclusion is, "We are jeopardizing our health by exporting banned pesticides." And we're asked to weaken that conclusion. If C is true, Mexico might get DDT from Canada even if we didn't sell it to them. And if that's true, then our tomatoes from Mexico might still have DDT on them, even if we didn't export the DDT to Mexico. Imagine you have 100 guns pointed at your face. If you give me one more gun, would you "significantly" increase your risk of being shot by doing so? I don't think so. If Mexico is already going to get DDT from Canada and douse our tomatoes with it, are we really "significantly" increasing our risk by exporting DDT to Mexico? Nope. So this answer, if true, would weaken the argument.

T50 - S2 - Q18 Asked by researchers to sort objects by shape, most toddlers in a large study had no trouble doing so. When subsequently told to sort by color, the toddlers seemed to have difficulty following the new rule and almost invariably persisted with their first approach. The researchers suggest such failures to adapt to new rules often result from insufficient development of the prefrontal cortex in toddlers. The cortex is essential for functions like adapting to new rules, yet is slow to mature, continuing to develop right into adolescence. Which one of the following is most supported by the information above? A Toddlers unable to sort objects by color tend to have a less developed prefrontal cortex than other children of the same age. B Only adolescents and adults can solve problems that require adapting to new rules. C Certain kinds of behavior on the part of toddlers may not be willfully disobedient. D The maturing of the prefrontal cortex is more important than upbringing in causing the development of adaptive behavior. E Skill at adapting to new situations is roughly proportional to the level of development of the prefrontal cortex.

C. The toddlers were asked to sort by shape, and they did it with no problem. Then they were asked to sort by color, and they kept sorting by shape. Researchers think this has something to do with their puny brains. The question says, "Which one of the following is most supported by the information above." This basically means, "Which one of the following must be true, based on the above." C has to be true. These kids, who are toddlers, didn't do what they were asked. But it's possible that they weren't being "willfully disobedient." They could have been confused by the new rule. So, based on just those toddlers, it *must be true* that sometimes, certain toddlers do certain things that "may not be" willfully disobedient. Note that this answer is very weakly stated (some kids, some behaviors, "may not"), which makes it a good candidate for a "must be true" type of question. *It's easier to prove weakly-stated answer choices.* This is our answer unless D or E really speaks to us.

T58 - S1 - Q22 When a society undergoes slow change, its younger members find great value in the advice of its older members. But when a society undergoes rapid change, young people think that little in the experience of their elders is relevant to them, and so do not value their advice. Thus, we may measure the rate at which a society is changing by measuring the amount of deference its younger members show to their elders. Which one of the following is an assumption on which the argument depends? A A society's younger members can often accurately discern whether that society is changing rapidly. B How much deference young people show to their elders depends on how much of the elders' experience is practically useful to them. C The deference young people show to their elders varies according to how much the young value their elders' advice. D The faster a society changes, the less relevant the experience of older members of the society is to younger members. E Young people value their elders' advice just insofar as the elders' experience is practically useful to them.

C. Think about it. Do you actually say "OK Boomer" out loud to your dad? Or roll your eyes in your grandma's face? I mean, maybe you do sometimes, as a joke. Certainly, you complain to your friends about them. But you can "show deference" to someone whose advice you don't value at all. Maybe it's in your best interest. Or maybe you just feel like being nice. The argument brought up a slightly new idea, "showing deference," in the conclusion. You're not allowed to bring up new shit in your conclusion. That's the entire point of this question. Necessary Assumption questions are about finding the one answer that the author must agree with. In other words, the correct answer is the one that, when false, will destroy the author's argument. The argument has conflated how much someone values advice with how much deference that person shows to the advice-giver. If C is false, the argument is ruined. That's why this is the answer.

T60 - S3 - Q23 A species in which mutations frequently occur will develop new evolutionary adaptations in each generation. Since species survive dramatic environmental changes only if they develop new evolutionary adaptations in each generation, a species in which mutations frequently occur will survive dramatic environmental changes. The flawed pattern of reasoning in which one of the following is most closely parallel to that in the argument above? A In a stone wall that is properly built, every stone supports another stone. Since a wall's being sturdy depends upon its being properly built, only walls that are composed entirely of stones supporting other stones are sturdy. B A play that is performed before a different audience every time will never get the same reaction from any two audiences. Since no plays are performed before the same audience every time, no play ever gets the same reaction from any two audiences. C A person who is perfectly honest will tell the truth in every situation. Since in order to be a morally upright person one must tell the truth at all times, a perfectly honest person will also be a morally upright person. D An herb garden is productive only if the soil that it is planted in is well drained. Since soil that is well drained is good soil, an herb garden is not productive unless it is planted in good soil. E A diet that is healthful is well balanced. Since a well-balanced diet includes fruits and vegetables, one will not be healthy unless one eats fruits and vegetables.

C. This argument confuses necessary for sufficient. It's shocking how often the LSAT does this. In "species survive dramatic environmental changes only if they develop new evolutionary adaptations in each generation," the "only if" introduces a necessary condition. If a species survives, we know for sure they have to have developed new evolutionary adaptations in each generation. But the conclusion thinks that these adaptations are sufficient to prove survival. In order to be morally upright it's necessary to tell the truth, but that doesn't mean that telling the truth is sufficient to prove you are morally upright. C is the answer.

T54 - S2 - Q14 The number of serious traffic accidents (accidents resulting in hospitalization or death) that occurred on Park Road from 1986 to 1990 was 35 percent lower than the number of serious accidents from 1981 to 1985. The speed limit on Park Road was lowered in 1986. Hence, the reduction of the speed limit led to the decrease in serious accidents. Which one of the following statements, if true, most weakens the argument? A The number of speeding tickets issued annually on Park Road remained roughly constant from 1981 to 1990. B Beginning in 1986, police patrolled Park Road much less frequently than in 1985 and previous years. C The annual number of vehicles using Park Road decreased significantly and steadily from 1981 to 1990. D The annual number of accidents on Park Road that did not result in hospitalization remained roughly constant from 1981 to 1990. E Until 1986 accidents were classified as "serious" only if they resulted in an extended hospital stay.

C. This argument says, "There's a correlation, therefore there has to be causation." (A happened in proximity to B, therefore A must have caused B.) This is one of the two most common flaws that are tested on the LSAT. I can see this one coming a mile away.To weaken this argument, I'm ideally looking for an answer that provides an alternative explanation of what caused B. I want an answer that says, "It wasn't the lowering of the speed limit that caused the reduction in serious accidents, it was something else." Anything else will do. Let's scout for that first, before considering other attacks. C could be it. If this is true, then maybe the reduction in accidents was caused by the fact that people just aren't using the road any more. If that's true, then there would be fewer accidents regardless if the speed limit was kept the same, or lowered, or raised for that matter. This is exactly what we were dreaming of.

T60 - S1 - Q11 Most universities today offer students a more in-depth and cosmopolitan education than ever before. Until recently, for example, most university history courses required only the reading of textbooks that hardly mentioned the history of Africa or Asia after the ancient periods, or the history of the Americas' indigenous cultures. The history courses at most universities no longer display such limitations. Which one of the following, if true, most strengthens the argument above? A) The history courses that university students find most interesting are comprehensive in their coverage of various periods and cultures. B) Many students at universities whose history courses require the reading of books covering all periods and world cultures participate in innovative study-abroad programs. C) The extent to which the textbooks of university history courses are culturally inclusive is a strong indication of the extent to which students at those universities get an in-depth and cosmopolitan education. D) Universities at which the history courses are quite culturally inclusive do not always have courses in other subject areas that show the same inclusiveness. E) University students who in their history courses are required only to read textbooks covering the history of a single culture will not get an in-depth and cosmopolitan education from these courses alone.

C. This feels like it could be a trap. Do we really know that the books are now more "culturally inclusive"? That term was never used. Still, adding Africa, Asia, and "American indigenous cultures" does sound pretty broad in scope. And I like that this answer connects what's going on in history departments to "in-depth and cosmopolitan education" overall. So even though I don't love this answer, I could pick it if everything else sucks.

T54 - S2 - Q18 Columnist: Taking a strong position on an issue makes one likely to misinterpret or ignore additional evidence that conflicts with one's stand. But in order to understand an issue fully, it is essential to consider such evidence impartially. Thus, it is best not to take a strong position on an issue unless one has already considered all important evidence conflicting with that position. The columnist's reasoning most closely conforms to which one of the following principles? A It is reasonable to take a strong position on an issue if one fully understands the issue and has considered the evidence regarding that issue impartially. B To ensure that one has impartially considered the evidence regarding an issue on which one has taken a strong position, one should avoid misinterpreting or ignoring evidence regarding that issue. C Anyone who does not understand an issue fully should avoid taking a strong position on it. D One should try to understand an issue fully if doing so will help one to avoid misinterpreting or ignoring evidence regarding that issue. E It is reasonable to take a strong position on an issue only if there is important evidence conflicting with that position.

C. Which one of these things do we know, for sure, the columnist believes? Which one of these five do we have evidence, from her statement, to support? In C, the columnist is saying "if you haven't considered the evidence, don't take a strong position." That's exactly what this says. It's not different, it's not extra, it's perfect.

T54 - S2 - Q17 Ethicist: The penalties for drunk driving are far more severe when the drunk driver accidentally injures people than when no one is injured. Moral responsibility for an action depends solely on the intentions underlying the action and not on the action's results. Therefore, legal responsibility, depending as it does in at least some cases on factors other than the agent's intentions, is different than moral responsibility. The claim that the penalties for drunk driving are far more severe when the drunk driver accidentally injures people than when no one is injured plays which one of the following roles in the ethicist's argument? A It is a premise offered in support of the claim that legal responsibility for an action is based solely upon features of the action that are generally unintended by the agent. B It is offered as an illustration of the claim that the criteria of legal responsibility for an action include but are not the same as those for moral responsibility. C It is offered as an illustration of the claim that people may be held morally responsible for an action for which they are not legally responsible. D It is a premise offered in support of the claim that legal responsibility depends in at least some cases on factors other than the agent's intentions. E It is a premise offered in support of the claim that moral responsibility depends solely on the intentions underlying the action and not on the action's result.

D.

T59 - S2 - Q23 This year a flood devastated a small river town. Hollyville, also a river town, responded with an outpouring of aid in which a majority of its residents participated, a proportion that far surpassed that of a few years ago when Hollyville sent aid to victims of a highly publicized earthquake. This year's circumstances were a reversal of last year's, when Hollyville itself was the scene of a deadly tornado and so the recipient rather than the supplier of emergency aid. The situation described above most closely conforms to which one of the following generalizations? A People are more likely to aid people they know than they are to aid strangers. B Those who have received aid are more likely to be in favor of government relief programs than are those who have not. C The amount of aid that victims of a disaster receive is unrelated to the extent to which the disaster is publicized. D Once a disaster has struck them, people are more likely to aid others in need than they were before the disaster. E People are more likely to aid those who have experienced a hardship similar to one they themselves have experienced than to aid those who have experienced a dissimilar hardship.

D. A question that asks for an answer that "closely conforms" to a situation prefers an answer that's narrowly tailored to that scenario. Anything different or extra is wrong. The correct answer should feel like "yeah, no shit—that's exactly what they were talking about." The good folks of Hollyville gave more this year, the year right after they themselves needed disaster aid, than they gave a few years ago. D, conforms more closely to the scenario we were given than any of the other answers.

T60 - S3 - Q17 Any food that is not sterilized and sealed can contain disease-causing bacteria. Once sterilized and properly sealed, however, it contains no bacteria. There are many different acceptable food-preservation techniques; each involves either sterilizing and sealing food or else at least slowing the growth of disease-causing bacteria. Some of the techniques may also destroy natural food enzymes that cause food to spoil or discolor quickly. If the statements above are true, which one of the following must be true? A All food preserved by an acceptable method is free of disease-causing bacteria. B Preservation methods that destroy enzymes that cause food to spoil do not sterilize the food. C Food preserved by a sterilization method is less likely to discolor quickly than food preserved with other methods. D Any nonsterilized food preserved by an acceptable method can contain disease-causing bacteria. E If a food contains no bacteria, then it has been preserved by an acceptable method.

D. Any food that is not sterilized and sealed can contain disease-causing bacteria. So if it's preserved with some other acceptable method, but not sterilized, then it might contain disease-causing bacteria. This answer must be true if the given facts are true.

T47 - S1 - Q11 In contemplating major purchases, businesses often consider only whether there is enough money left from monthly revenues after paying monthly expenses to cover the cost of the purchase. But many expenses do not occur monthly; taking into account only monthly expenses can cause a business to overexpand. So the use of a cash-flow statement is critical for all businesses. Which one of the following, if true, most strengthens the argument? A Only a cash-flow statement can accurately document all monthly expenses. B Any business that has overexpanded can benefit from the use of a cash-flow statement. C When a business documents only monthly expenses it also documents only monthly revenue. D A cash-flow statement is the only way to track both monthly expenses and expenses that are not monthly. E When a business takes into account all expenses, not just monthly ones, it can make better decisions.

D. Ideally, when you arrived at the end of this argument you said to yourself, "Cash flow statement? What the **** is a cash flow statement? Where did that come from?!" If you are able to recognize that the term "cash flow statement" appeared nowhere else in the argument besides the conclusion, you are definitely on the right track. That's just not a legitimate argumentative strategy on the LSAT. The concepts must connect together. If you're going to tell me I need to have a cash flow statement, you better well provide some ******* evidence about why cash flow statements are necessary. We're asked to strengthen the argument, so I'd love an answer that protects against my objection by connecting the facts to the conclusion. Something like, "The cash flow statement is the best/only/easiest way to plan for irregular/unplanned expenses" would be a very nice answer. Aha! D matches our prediction. If this is true, then a cash flow statement is the only way (i.e., it's necessary, i.e., it's "critical") for tracking non-monthly expenses. (And monthly expenses as well, although that's irrelevant.) This answer does more than we need it to, but that's just fine if we're trying to strengthen the argument.

T59 - S2 - Q20 Quality control investigator: Upon testing samples of products from our supplier that were sent by our field inspectors from various manufacturing locations, our laboratory discovered that over 20 percent of the samples were defective. Since our supplier is contractually required to limit the rate of defects among items it manufactures for us to below 5 percent, it has violated its contract with us. The reasoning in the quality control investigator's argument is flawed in that the argument A bases its conclusion on too small a sample of items tested by the laboratory B presumes, without providing justification, that the field inspectors were just as likely to choose a defective item for testing as they were to choose a nondefective item C overlooks the possibility that a few of the manufacturing sites are responsible for most of the defective items D overlooks the possibility that the field inspectors tend to choose items for testing that they suspect are defective E presumes, without providing justification, that the field inspectors made an equal number of visits to each of the various manufacturing sites of the supplier

D. If the inspectors are intentionally choosing defective items, then their 20% sampling rate is biased in favor of errors. This is a fair accusation, because Mr. QCI did "overlook this possibility" and this possibility is a big potential problem for his case. This is the correct answer.

T57 - S2 - Q12 Criminologist: The main purpose of most criminal organizations is to generate profits. The ongoing revolutions in biotechnology and information technology promise to generate enormous profits. Therefore, criminal organizations will undoubtedly try to become increasingly involved in these areas. The conclusion of the criminologist's argument is properly inferred if which one of the following is assumed? A If an organization tries to become increasingly involved in areas that promise to generate enormous profits, then the main purpose of that organization is to generate profits. B At least some criminal organizations are or will at some point become aware that the ongoing revolutions in biotechnology and information technology promise to generate enormous profits. C Criminal organizations are already heavily involved in every activity that promises to generate enormous profits. D Any organization whose main purpose is to generate profits will try to become increasingly involved in any technological revolution that promises to generate enormous profits. E Most criminal organizations are willing to become involved in legal activities if those activities are sufficiently profitable.

D. The question asks us to identify a sufficient condition, and our task is to make this horrible argument make sense. The best way to do that is to reverse my last statement in the previous paragraph. If anyone who pursues money will pursue allsources of money, then Tony Soprano, who pursues money, will have to pursue all sources of money, including IT and biotech. So I'm looking for, "Anyone who pursues money will pursue all sources of money," or even something broader than that, like, "Anyone who pursues anything will pursue everything." When we're looking for a sufficient condition, it's impossible for the answer to be too strong or too broad. D basically says, "If you want money at all, you'll pursue every source of money." If this were true, then Tony would have to put on his lab coat. This is the answer.

T57 - S2 - Q20 In the past, when there was no highway speed limit, the highway accident rate increased yearly, peaking a decade ago. At that time, the speed limit on highways was set at 90 kilometers per hour (kph) (55 miles per hour). Every year since the introduction of the highway speed limit, the highway accident rate has been at least 15 percent lower than that of its peak rate. Thus, setting the highway speed limit at 90 kph (55 mph) has reduced the highway accident rate by at least 15 percent. Which one of the following, if true, most seriously weakens the argument? A In the years prior to the introduction of the highway speed limit, many cars could go faster than 90 kph (55 mph). B Ten years ago, at least 95 percent of all automobile accidents in the area occurred on roads with a speed limit of under 80 kph (50 mph). C Although the speed limit on many highways is officially set at 90 kph (55 mph), most people typically drive faster than the speed limit. D Thanks to changes in automobile design in the past ten years, drivers are better able to maintain control of their cars in dangerous situations. E It was not until shortly after the introduction of the highway speed limit that most cars were equipped with features such as seat belts and airbags designed to prevent harm to passengers.

D. If we can prove that anything else other than the speed limit caused the decline in accidents (less traffic, perhaps? better policing? safer cars?), then we have seriously weakened the idea that the speed limit caused the decline in accidents. We're asked to weaken the argument, and my prediction is, "What if the Martians are responsible?" This isn't literally the Martians, but it is the Martians, metaphorically. If D is true, then something else is at least partially responsible for the decline in the accident rate. Which makes the argument's conclusion look pretty silly.

T59 - S2 - Q18 Contrary to Malthus's arguments, human food-producing capacity has increased more rapidly than human population. Yet, agricultural advances often compromise biological diversity. Therefore, Malthus's prediction that insufficient food will doom humanity to war, pestilence, and famine will likely be proven correct in the future, because a lack of biodiversity will eventually erode our capacity to produce food. The statement that human food-producing capacity has increased more rapidly than human population plays which one of the following roles in the argument? A It is a hypothesis the argument provides reasons for believing to be presently false. B It is a part of the evidence used in the argument to support the conclusion that a well-known view is misguided. C It is an observation that the argument suggests actually supports Malthus's position. D It is a general fact that the argument offers reason to believe will eventually change. E It is a hypothesis that, according to the argument, is accepted on the basis of inadequate evidence.

D. On Role questions, it's best to keep it simple to start. Ask yourself, "Is it the conclusion? Or is it evidence for that conclusion? Or is it something else?" Here, it's definitely not the conclusion. The conclusion was "Malthus will eventually be proven correct." And it's also not evidence in favor of that conclusion. There's no way that the argument can possibly be saying "human food-producing capacity has increased more rapidly than the human population, therefore Malthus will eventually be proven correct." Malthus says we're going to starve! The fact that so far, human food-producing capacity has grown more rapidly than the human population does not support the conclusion that we're going to starve—it suggests the exact opposite of that. Heading into the answer choices, my prediction is "something else." I know it's not the conclusion, and I know it's not support for the conclusion. It's something else. Keep it simple. The author acknowledges that we haven't starved yet, but offers reasons to believe that this will change, then concludes that it will change. D is the answer.

T60 - S1 - Q9 In one study of a particular plant species, 70 percent of the plants studied were reported as having patterned stems. In a second study, which covered approximately the same geographical area, only 40 percent of the plants of that species were reported as having patterned stems. Which one of the following, if true, most helps to resolve the apparent discrepancy described above? A) The first study was carried out at the time of year when plants of the species are at their most populous. B) The first study, but not the second study, also collected information about patterned stems in other plant species. C) The second study included approximately 15 percent more individual plants than the first study did. D) The first study used a broader definition of "patterned." E) The focus of the second study was patterned stems, while the first study collected information about patterned stems only as a secondary goal.

D. The first study used a broader definition of "patterned," so that 70 percent of observed plants met the broader definition of what "patterned" meant. The second study narrowed the definition of "patterned," so that only 40 percent of the observed plants met the definition. If this is true, then there doesn't have to have been any change in the underlying plants themselves; this just says that the survey methodology was changed between studies in a way that explains the results perfectly. This is the answer.

T60 - S3 - Q11 A mass of "black water" containing noxious organic material swept through Laurel Bay last year. Some scientists believe that this event was a naturally occurring but infrequent phenomenon. The black water completely wiped out five species of coral in the bay, including mounds of coral that were more than two centuries old. Therefore, even if this black water phenomenon has struck the bay before, it did not reach last year's intensity at any time in the past two centuries. Which one of the following is an assumption required by the argument? A Masses of black water such as that observed last summer come into the bay more frequently than just once every two centuries. B Every species of coral in the bay was seriously harmed by the mass of black water that swept in last year. C The mass of black water that swept through the bay last year did not decimate any plant or animal species that makes use of coral. D The mounds of centuries-old coral that were destroyed were not in especially fragile condition just before the black water swept in last year. E Older specimens of coral in the bay were more vulnerable to damage from the influx of black water than were young specimens.

D. The logic is basically like this: If 200-year-old coral were destroyed by this recent black water phenomenon (but not whatever black water events hit in the last 200 years) then the last 200 years of black water phenomena can't have been as intense as the most recent one that killed the coral. So the argument has necessarily assumed that the coral couldn't have survived a previous, more intense black water event. Boom perfect. If this is false it's "the coral were in real bad shape due to previous events (including, perhaps, previous devastating black water events)." This is the answer because when it's false, the argument fails.

T46 - S2 - Q13 Companies wishing to boost sales of merchandise should use in-store displays to catch customers' attention. According to a marketing study, today's busy shoppers have less time for coupon-clipping and pay little attention to direct-mail advertising; instead, they make two-thirds of their buying decisions on the spot at the store. Which one of the following is an assumption that the argument requires? A Companies are increasingly using in-store displays to catch customers' attention. B Coupons and direct-mail advertising were at one time more effective means of boosting sales of merchandise than they are now. C In-store displays are more likely to influence buying decisions made on the spot at the store than to influence other buying decisions. D In-store displays that catch customers' attention increase the likelihood that customers will decide on the spot to buy the company's merchandise. E Many of today's shoppers are too busy to pay careful attention to in-store displays.

D. We're asked to find an assumption that the argument requires, which makes this a Necessary Assumption question. Necessary Assumption questions are closely linked to Weaken questions. I think the correct answer might be a defense against one of our objections above. Something like, "People do pay attention to in-store displays" would defend against our first and second objections, above. And I think that's a Necessary Assumption of the argument, because if it is not true, then the argument would fail. Let's see: Yes. D is what we predicted. This is necessary, because if it is untrue the argument would be totally destroyed.

T60 - S3 - Q15 Technologically, it is already possible to produce nonpolluting cars that burn hydrogen rather than gasoline. But the national system of fuel stations that would be needed to provide the hydrogen fuel for such cars does not yet exist. However, this infrastructure is likely to appear and grow rapidly. A century ago no fuel-distribution infrastructure existed for gasoline-powered vehicles, yet it quickly developed in response to consumer demand. Which one of the following most accurately expresses the conclusion drawn in the argument? A It is already technologically possible to produce nonpolluting cars that burn hydrogen rather than gasoline. B The fuel-distribution infrastructure for hydrogen- powered cars still needs to be created. C If a new kind of technology is developed, the infrastructure needed to support that technology is likely to quickly develop in response to consumer demands. D The fuel-distribution infrastructure for hydrogen-powered cars is likely to appear and grow rapidly. E Hydrogen-powered vehicles will be similar to gasoline-powered vehicles with regard to the amount of consumer demand for their fuel-distribution infrastructure.

D. Here, we don't find any of those obvious premise or conclusion indicators. But we do find a prediction about the future: "infrastructure is likely to appear and grow rapidly." We also find a reason to believe that this prediction might come true: "A century ago no fuel-distribution infrastructure existed for gasoline- powered vehicles, yet it quickly developed in response to consumer demand." At some point in every argument, the speaker will provide a reason to believe the thing they are trying to prove. In this argument, that happens exactly once. The premise in this argument is the thing about fuel-distribution infrastructure for gas-powered vehicles. The conclusion that this premise supports is "infrastructure for hydrogen cars is likely to appear and grow rapidly." Yup. This is lifted directly from the argument and perfectly matches my prediction.

T57 - S2 - Q5 Studies reveal that most people select the foods they eat primarily on the basis of flavor, and that nutrition is usually a secondary concern at best. This suggests that health experts would have more success in encouraging people to eat wholesome foods if they emphasized how flavorful those foods truly are rather than how nutritious they are. Which one of the following, if true, most strengthens the argument above? A Most people currently believe that wholesome foods are more flavorful, on average, than unwholesome foods are. B Few people, when given a choice between foods that are flavorful but not nutritious and foods that are nutritious but not flavorful, will choose the foods that are nutritious but not flavorful. C Health experts' attempts to encourage people to eat wholesome foods by emphasizing how nutritious those foods are have been moderately successful. D The studies that revealed that people choose the foods they eat primarily on the basis of flavor also revealed that people rated as most flavorful those foods that were least nutritious. E In a study, subjects who were told that a given food was very flavorful were more willing to try the food and more likely to enjoy it than were subjects who were told that the food was nutritious.

E.

T58 - S1 - Q20 A person with a type B lipid profile is at much greater risk of heart disease than a person with a type A lipid profile. In an experiment, both type A volunteers and type B volunteers were put on a low-fat diet. The cholesterol levels of the type B volunteers soon dropped substantially, although their lipid profiles were unchanged. The type A volunteers, however, showed no benefit from the diet, and 40 percent of them actually shifted to type B profiles. If the information above is true, which one of the following must also be true? A In the experiment, most of the volunteers had their risk of heart disease reduced at least marginally as a result of having been put on the diet. B People with type B lipid profiles have higher cholesterol levels, on average, than do people with type A lipid profiles. C Apart from adopting the low-fat diet, most of the volunteers did not substantially change any aspect of their lifestyle that would have affected their cholesterol levels or lipid profiles. D The reduction in cholesterol levels in the volunteers is solely responsible for the change in their lipid profiles. E For at least some of the volunteers in the experiment, the risk of heart disease increased after having been put on the low-fat diet.

E.

T59 - S2 - Q16 All poets, aside from those who write only epigrams, have wit. All lyrical composers are poets. Azriel does not write epigrams, though he is a lyrical composer. So Azriel has wit. The pattern of reasoning in which one of the following is most similar to that in the argument above? A All squeeze toys, except those designed for cats, are safe for infants. All squeeze toys are sold prewrapped. This item is not designed for cats, and it is sold prewrapped. So it must be safe for infants. B Aside from the dogcatcher and the police chief, all of the politicians in town are lawyers. All of the politicians in town have websites. Sal is a politician in town, but is neither the dogcatcher nor the police chief. Since Sal is a politician in town he must have a website. C All visas are assigned by this office, except for those that are issued through diplomatic channels. All visit permits are visas. Thus, the visit permit in Will's passport was assigned through diplomatic channels. D All of this store's winter garments are on sale, except for the designer clothes. None of the shirts in this store are designer clothes. This shirt, therefore, since it is on sale, is a winter garment. E All residential buildings are subject to the original fire code, except for those built last year. All townhouses are residential buildings. Bloom House was not built last year, and it is a townhouse, so it is subject to the original fire code.

E.

T50 - S2 - Q11 Cynthia: Corporations amply fund research that generates marketable new technologies. But the fundamental goal of science is to achieve a comprehensive knowledge of the workings of the universe. The government should help fund those basic scientific research projects that seek to further our theoretical knowledge of nature. Luis: The basic goal of government support of scientific research is to generate technological advances that will benefit society as a whole. So only research that is expected to yield practical applications in fields such as agriculture and medicine ought to be funded. Cynthia's and Luis's statements provide the most support for the contention that they would disagree with each other about the truth of which one of the following statements? A The government should help fund pure theoretical research because such research might have unforeseen practical applications in fields such as agriculture and medicine. B A proposed study of the effects of chemical fertilizers on crops, for the purpose of developing more-resistant and higher-yielding breeds, should not receive government funding. C Although some research projects in theoretical science yield practical benefits, most do not, and so no research projects in theoretical science should be funded by the government. D Research for the sole purpose of developing new technologies ought to be financed exclusively by corporations. E Knowledge gained through basic scientific research need not be expected to lead to new and useful technologies in order for the research to merit government funding.

E. Cynthia agrees with E—she wants the government to fund theoretical research solely in the pursuit of theoretical knowledge. Luis disagrees with this—Luis doesn't want the government to fund anything that's not expected to yield practical benefits. This is the only answer where we have a clear yes from one speaker and a clear no from the other. So this is the answer.

T58 - S1 - Q8 Working residents of Springfield live, on average, farther from their workplaces than do working residents of Rorchester. Thus, one would expect that the demand for public transportation would be greater in Springfield than in Rorchester. However, Springfield has only half as many bus routes as Rorchester. Each of the following, if true, contributes to a resolution of the apparent discrepancy described above EXCEPT: A Three-fourths of the Springfield workforce is employed at the same factory outside the city limits. B The average number of cars per household is higher in Springfield than in Rorchester. C Rorchester has fewer railway lines than Springfield. D Buses in Springfield run more frequently and on longer routes than in Rorchester. E Springfield has a larger population than Rorchester does.

E. E makes it harder to understand why Springfield has fewer routes. This is the exact opposite of my prediction, actually. I was all like "maybe Springfield has fewer people!" This answer's all like "nah bro—Springfield has more people." Not only does this not solve the paradox, this actually makes the paradox worse. It's a dream answer for a Paradox... EXCEPT.

T54 - S2 - Q22 Student: The publications of Professor Vallejo on the origins of glassblowing have reopened the debate among historians over whether glassblowing originated in Egypt or elsewhere. If Professor Vallejo is correct, there is insufficient evidence for claiming, as most historians have done for many years, that glassblowing began in Egypt. So, despite the fact that the traditional view is still maintained by the majority of historians, if Professor Vallejo is correct, we must conclude that glassblowing originated elsewhere. Which one of the following is an error in the student's reasoning? A It draws a conclusion that conflicts with the majority opinion of experts. B It presupposes the truth of Professor Vallejo's claims. C It fails to provide criteria for determining adequate historical evidence. D It mistakes the majority view for the traditional view. E It confuses inadequate evidence for truth with evidence for falsity.

E. Fairly common flaw here. The student is wrong, because disproving an argument doesn't disprove the conclusion of that argument. For example, watch this conversation: Speaker 1: "I think John got mauled by a bear yesterday, therefore John is dead." Speaker 2: "But John didn't get mauled by a bear yesterday. Therefore John is certainly alive." Speaker 1: "No you *******, he's not certainly alive. If John didn't get mauled by a bear, then you've defeated my argument. I admit that my conclusion wasn't justified. And I agree that John is probably alive, but you haven't proven that he's alive, so stop saying ****ing 'certainly.' For all we know, a pack of rabid raccoons could have eaten his face off this morning." We're asked to identify an error in the student's reasoning, which we've already done. The student has confused defeating an argument with proving the opposite of the argument's conclusion. Yep. E is another way of saying, "The student thinks that if the historians' proof is bad, then the exact opposite of what the historians claim must be true." That's exactly what we were looking for.

T59 - S2 - Q5 Gilbert: This food label is mistaken. It says that these cookies contain only natural ingredients, but they contain alphahydroxy acids that are chemically synthesized by the cookie company at their plant. Sabina: The label is not mistaken. After all, alphahydroxy acids also are found occurring naturally in sugarcane. Which one of the following, if true, would most strengthen Sabina's argument? A The cookie company has recently dropped alphahydroxy acids from its cookie ingredients. B Not all chemicals that are part of the manufacturing process are ingredients of the cookies. C The label was printed before the cookie company decided to switch from sugarcane alphahydroxy acids to synthesized ones. D Many other foods advertising all natural ingredients also contain some ingredients that are chemically synthesized. E All substances except those that do not occur naturally in any source are considered natural.

E. Gilbert doesn't want the food label to say "these cookies contain only natural ingredients" when they contain alphahydroxy acids that are chemically synthesized in a factory. Seems pretty legit, right? How the hell can these be "natural" if they're full of alphahydroxy acids that were chemically synthesized in a factory? Sabina disagrees. She doesn't argue about whether the cookies contain alphahydroxy acids, or about whether these acids are chemically synthesized in a factory—she seems willing to grant these points. But she adds a bit of new information that she thinks gets the Keebler elves off the hook: Alphahydroxy acids appear naturally in sugarcane. Essentially, what Sabina seems to be saying is that as long as something appears naturally, it's cool to put factory-synthesized versions of that same thing into your cookies and call them "all natural." Not only does E help Sabina, it actually makes her win her argument. When you remove the double-negative, this answer says "if it appears anywhere in nature, it's 'natural.'" That's exactly what Sabina was arguing.

T59 - S2 - Q7 Columnist: It has been noted that attending a live musical performance is a richer experience than is listening to recorded music. Some say that this is merely because we do not see the performers when we listen to recorded music. However, there must be some other reason, for there is relatively little difference between listening to someone read a story over the radio and listening to someone in the same room read a story. Which one of the following most accurately expresses the role played in the argument by the observation that attending a live musical performance is a richer experience than is listening to recorded music? A It is what the columnist's argument purports to show. B It is the reason given for the claim that the columnist's argument is attempting to undermine. C It is what the columnist's argument purports to explain. D It is what the columnist's argument purports to refute. E It is what the position that the columnist tries to undermine is purported to explain.

E. So like, "attending a live musical performance is a richer experience than is listening to recorded music" is the thing that we're talking about explaining, right? It's not the conclusion; the conclusion is basically "idk, there must be some other reason." The premise is the whole radio analogy. So "attending a live musical performance is a richer experience than is listening to recorded music" is the thing that the conclusion says is not, yet, explained—the argument shits on one explanation, but fails to itself offer a competing explanation. The "position the columnist tries to undermine" is "we can see the performers." "We can see the performers" is purported to explain "attending a live musical performance is a richer experience than is listening to recorded music." So, "attending a live musical performance is a richer experience than is listening to recorded music" is "what the position that the columnist tries to undermine is purported to explain."

T57 - S2 - Q17 Poetry journal patron: Everybody who publishes in The Brick Wall Review has to agree in advance that if a poem is printed in one of its regular issues, the magazine also has the right to reprint it, without monetary compensation, in its annual anthology. The Brick Wall Review makes enough money from sales of its anthologies to cover most operating expenses. So, if your magazine also published an anthology of poems first printed in your magazine, you could depend less on donations. After all, most poems published in your magazine are very similar to those published in The Brick Wall Review. Which one of the following, if true, most weakens the patron's argument? A Neither The Brick Wall Review nor the other magazine under discussion depends on donations to cover most operating expenses. B Many of the poets whose work appears in The Brick Wall Review have had several poems rejected for publication by the other magazine under discussion. C The only compensation poets receive for publishing in the regular issues of the magazines under discussion are free copies of the issues in which their poems appear. D The Brick Wall Review depends on donations to cover most operating expenses not covered by income from anthology sales. E The Brick Wall Review's annual poetry anthology always contains a number of poems by famous poets not published in the regular issues of the magazine.

E. The conclusion is, "You could depend less on donations if you did an anthology." The evidence for this assertion is the example of another, similar magazine (The Brick Wall Review) that does anthologies and thereby covers most of its expenses. We are asked to weaken this argument. At first I didn't think that the fame of the poets could be relevant, and wanted to eliminate answer E. But fame is really just a distraction. The recommendation in the conclusion of the argument was, "Make an anthology of poems first printed in your magazine... you'll make money." If E is true, then the anthology used for comparison may have made money because it printed a lot of poems not first published in regular issues of the magazine. So the recommendation to reprint a lot of stuff that was already printed in the magazine is not necessarily reliable, and this is our answer. Again, this is a really tricky answer because it actually doesn't matter whether or not they are famous. What matters is whether or not they were included in the regular issues of the magazine.

T46 - S2 - Q8 The laboratory experiment, the most effective method for teaching science, is disappearing from most secondary school curricula, and students are now simulating experiments with computers. This trend should be stopped. It results in many students' completing secondary school and going on to a university without knowing how to work with laboratory equipment. Which one of the following, if true, most weakens the argument? A Scientific knowledge is changing so rapidly it is difficult for secondary schools to keep up without using computers. B In some secondary schools, teachers conduct laboratory experiments while students observe. C Computers have proven to be a valuable tool for teaching secondary school students scientific terminology. D Secondary schools and universities across the nation have put a great deal of money into purchasing computers. E University students can learn science effectively without having had experience in working with laboratory equipment.

E. The conclusion states that we should stop transitioning away from lab experiments. Why? Without lab experiments, students won't know how to use lab equipment before they go to university. We're asked to weaken this passage. We want to show that the trend doesn't need to stop. Therefore, we need to show that understanding how to use laboratory equipment is irrelevant. They don't need it for university. E is the best weakener. It tells us that learning how to use lab equipment isn't essential for moving onto university. While it's most effective to teach via lab experiments in high school, university students can get by without understanding lab equipment.

T58 - S1 - Q16 Corporate businesses, like species, must adapt to survive. Businesses that are no longer efficient will become extinct. But sometimes a business cannot adapt without changing its core corporate philosophy. Hence, sometimes a business can survive only by becoming a different corporation. Which one of the following is an assumption required by the argument? A No business can survive without changing its core corporate philosophy. B As a business becomes less efficient, it invariably surrenders its core corporate philosophy. C Different corporations have different core corporate philosophies. D If a business keeps its core corporate philosophy intact, it will continue to exist. E A business cannot change its core corporate philosophy without becoming a different corporation.

E. The instructions at the top of each section of LR include, "In answering the questions, you should not make assumptions that are by commonsense standards implausible, superfluous, or incompatible with the passage." If Shelby thinks the two instances of "sometimes" in the passage might refer to different times, that's just not commonsense, and it's implausible at the least. These arguments are bad, but they're not that bad. They're trying to make an argument, not spouting complete gibberish. I'll address Shelby's second comment when I reach the correct answer below. Necessary Assumption questions ask us to identify the answer the author must agree with. Put another way, the correct answer, when false, will destroy the argument. The author has to agree with E, that's why it's the answer. Shelby said, "I wish the answer said 'sometimes,'" and well, this certainly could have said "sometimes" and still been correct. But it didn't need to. The author said "sometimes a business cannot adapt without changing its core corporate philosophy," then immediately concluded that, therefore, these businesses "can survive only by becoming a different corporation." This means, necessarily, that the author must agree that any time you change your core corporate philosophy you also become a different corporation. If that's not true all the time, then companies that have to change core corporate philosophies can survive without becoming different corporations—the argument is destroyed.

T58 - S1 - Q24 The Kiffer Forest Preserve, in the northernmost part of the Abbimac Valley, is where most of the bears in the valley reside. During the eight years that the main road through the preserve has been closed the preserve's bear population has nearly doubled. Thus, the valley's bear population will increase if the road is kept closed. Which one of the following, if true, most undermines the argument? A Most of the increase in the preserve's bear population over the past eight years is due to migration. B Only some of the increase in the preserve's bear population over the past eight years is due to migration of bears from other parts of the Abbimac Valley. C Only some of the increase in the preserve's bear population over the past eight years is due to migration of bears from outside the Abbimac Valley. D The bear population in areas of the Abbimac Valley outside the Kiffer Forest Preserve has decreased over the past eight years. E The bear population in the Abbimac Valley has remained about the same over the past eight years.

E. The problem in this argument is that the evidence is about the preserve while the conclusion is about the valley. It's going from part to whole. In other words, what happened to the preserve might not necessarily happen to the valley, especially if the preserve's population increased by taking bears away from the rest of the valley. In that case, the valley's population stayed the same. E is correct because it doesn't mess around with the preserve or the areas outside the preserve. It cuts to the chase by focusing solely on the valley as a whole, which is what the conclusion is all about. If the bear population in the valley as a whole has stayed the same for the last eight years, then why should we believe that keeping the road closed will now lead to an increase of bears? Sure, it might lead to an increase of bears within the preserve. But the valley as a whole has been unaffected, and thus will probably continue to remain unaffected.

T57 - S2 - Q24 Sometimes one reads a poem and believes that the poem expresses contradictory ideas, even if it is a great poem. So it is wrong to think that the meaning of a poem is whatever the author intends to communicate to the reader by means of the poem. No one who is writing a great poem intends it to communicate contradictory ideas. Which one of the following is an assumption on which the argument depends? A Different readers will usually disagree about what the author of a particular poem intends to communicate by means of that poem. B If someone writes a great poem, he or she intends the poem to express one primary idea. C Readers will not agree about the meaning of a poem if they do not agree about what the author of the poem intended the poem to mean. D Anyone reading a great poem can discern every idea that the author intended to express in the poem. E If a reader believes that a poem expresses a particular idea, then that idea is part of the meaning of the poem.

E. We're asked to identify a necessary assumption of the argument. I think the answer could be something like, "The reader's opinion sometimes matters when ascribing meaning to a poem," because if that statement is not true it would become, "The reader's opinion is never relevant when ascribing meaning to a poem," which would completely devastate the argument. That's how I know, "The reader's opinion sometimes matters when ascribing meaning to a poem," is a Necessary Assumption of the argument. E) is the closest thing to our prediction. If this answer is untrue, it becomes, "A reader's belief about the meaning of a poem is not part of the meaning of the poem." And if that's true, then the entire argument would just fall apart. So E is a necessary assumption of the argument, and it's our answer.

T60 - S1 - Q23 Libel is defined as damaging the reputation of someone by making false statements. Ironically, strong laws against libel can make it impossible for anyone in the public eye to have a good reputation. For the result of strong libel laws is that, for fear of lawsuits, no one will say anything bad about public figures. Which one of the following principles, if valid, most helps to justify the reasoning in the argument? A The absence of laws against libel makes it possible for everyone in the public eye to have a good reputation. B Even if laws against libel are extremely strong and rigorously enforced, some public figures will acquire bad reputations. C If one makes statements that one sincerely believes, then those statements should not be considered libelous even if they are in fact false and damaging to the reputation of a public figure. D In countries with strong libel laws, people make negative statements about public figures only when such statements can be proved. E Public figures can have good reputations only if there are other public figures who have bad reputations.

E. Premise: Libel is defined as damaging the reputation of someone by making false statements. Premise: The result of strong libel laws is that, for fear of lawsuits, no one will say anything bad about public figures. Conclusion: Strong laws against libel can make it impossible for anyone in the public eye to have a good reputation. Okay so we have a definition of libel (making damaging false statements about someone) and a consequence of strong libel laws (people will be afraid to talk shit about public figures). From these facts, the author wants to reach the conclusion that strong libel laws can make it impossible for anyone in the public eye to have a good reputation. Why would widespread reluctance to talk shit about public figures lead to a lack of good reputations among public figures? We can't think anybody's cool if we don't think other people are dicks? That's what they're saying, right? Bingo. My prediction was "we can't think anybody is cool unless we think other people are dicks." That's what this says, that's why this is the answer.

T60 - S1 - Q18 Bus driver: Had the garbage truck not been exceeding the speed limit, it would not have collided with the bus I was driving. I, on the other hand, was abiding by all traffic regulations—as the police report confirms. Therefore, although I might have been able to avoid the collision had I reacted more quickly, the bus company should not reprimand me for the accident. Which one of the following principles, if valid, most helps to justify the reasoning in the bus driver's argument? A If a vehicle whose driver is violating a traffic regulation collides with a vehicle whose driver is not, the driver of the first vehicle is solely responsible for the accident. B A bus company should not reprimand one of its drivers whose bus is involved in a collision if a police report confirms that the collision was completely the fault of the driver of another vehicle. C Whenever a bus driver causes a collision to occur by violating a traffic regulation, the bus company should reprimand that driver. D A company that employs bus drivers should reprimand those drivers only when they become involved in collisions that they reasonably could have been expected to avoid. E When a bus is involved in a collision, the bus driver should not be reprimanded by the bus company if the collision did not result from the bus driver's violating a traffic regulation.

E. The biggest weakness in the argument is that there is nothing to justify the idea that the bus company "should not reprimand" under these circumstances. It's a giant hole in the argument. To prove our case, we simply must connect the "should not reprimand" part in the conclusion to the facts that we already have. There are several ways to do this, since we have several facts. Imagine how much stronger the case would be if the law said, "Any time some-one else was speeding, you should not be reprimanded." Or, "Any time you would have avoided an accident if the other party hadn't been speeding, you should not be reprimanded." Or, "Any time you were not yourself speeding, you should not be reprimanded." Or, "Any time you were obeying the traffic regulations you should not be reprimanded." Any one of these laws would take evidence we already have and connect it, directly, to our desired conclusion. If any of these four things were the law, we would be sure to win our case. Since our driver didn't violate any traffic regulations, the collision cannot have resulted from our driver's violating a traffic regulation. So if E is the rule, then our bus driver can't be reprimanded. Which is our desired outcome. This one connects our facts more directly to our conclusion than A did.


Conjuntos de estudio relacionados

Fundamentals Week 2 Test 1 Material

View Set

Comptia 220-801 12.6.10 Practice Test Questions

View Set

CH 12 Transport of Proteins Into Mitochondria and Chloroplasts

View Set